NBME 11 Help!

This forum made possible through the generous support of SDN members, donors, and sponsors. Thank you.

britesky89

Full Member
7+ Year Member
Joined
Apr 6, 2015
Messages
65
Reaction score
25
Hi guys, I need help with a some questions! I went through a lot of the previous threads about NBME 11 but I was still confused about these questions.

"3 week old newborn, recurrent vomiting after feeding since birth and fever. Abdominal exam shows a firm 1-2 cm mobile mass in the epigastrium to the right of the midline. Assuming this condition has a lower threshold in males, which of the following relatives of this patient is at greatest risk for also developing this disorder?"
A. Brother, if newborn is female (correct)
B. Brother, if newborn is male
C. Fraternal twin, if newborn is male (WRONG)
D. Sister, if newborn is female
E. Sister, if newborn is male

So the reasoning is that the defect has an "EASIER" time being expressed in boys, thus for it to be expressed in a females means it would have very "STRONG GENETICS". But if the fraternal twin happens to be male, wouldn't that be a better answer, because its a more direct illustration of how pyloric stenosis has an easier time being expressed in BOYS?

"Patient has a loss of the pupillary light reflex in the LEFT eye when light is shown in EITHER eye. Where is the lesion?" I've attached the picture below.
I think this is what the letters are pointing at. Please correct me if I am wrong.
A. Right optic nerve
B. Optic chiasm (WRONG)
C. Left optic nerve
D. Right optic tract
E. Left optic tract
F. Right occulumotor nerve
G. Left occulomotor nerve (is this right, b/c pupillary reflex depends on CNIII? please explain the pathway)

The "EITHER" eye really threw me off. If it blocked the Left optic nerve then the right occulmotor nerve could induce the pupillary reflex via the Edinger Westphal nucleus (consentual light reflex). This is so confusing! please help!


"Study is conducted comparing total serum cholesterol among three groups of 50 subjects each according to age: Group A: children ages 6-10 yrs; Group B: adolescents ages 12-16 yrs; Group C: young adults ages 21-35.
Fasting serum cholesterol is measured in all subjects. Which of the following statistical tests is most appropriate to compare the serum total cholesterol concentration results among Groups A, B, and C?
A. Analysis of Variance (right?)
B. Chi-square test (WRONG)
C. Independent t-test
D. Multiple logistic test
E. Paired t-test
F. Pearson Correlation test

I thought it was comparing total cholesterol was a CATEGORICAL subject. But I guess we're supposed to assume that since they're comparing "TOTAL SERUM CHOLESTEROL" that they are going to use MEAN values in their analysis rather than PERCENTAGES (categorical)??

"28 yo F with BMI of 32 kg/m2, who has a sedentary lifestyle, wants to lose weight. She consumes 1800 calories per day. Assume 3500 calories per pound & brisk walking consumes 500 calories per hour. To help her lose 2 lbs per week, what would be the best plan?"
Calories per day Brisk Walking Daily
A. Decrease by 100 1/2 hr
B. Decrease by 100 1 hour
C. Decrease by 500 1/2 hour
D. Decrease by 500 1 hour (CORRECT)
E. No change 1/2 hour
F. No change 1 hour

I don't understand how we're supposed to reason through these kinds of questions. All I could think of was plugging in all the values to check if they worked. Of course, that wasted a lot of time; and I just guessed and moved on. Even then, I wasted too much time on this question and it really left me crunched for time for the rest of the block :inpain:. Can someone please explain the thought process to tackling such a question?


"54 y.o woman with 5 cm coin lesion in right upper lobe of lung. proliferation of irregularly shaped glands with hyperchromatic and pleomorphic nuclei invading the pulmonary parenchyma and lymphovascular spaces? Best described as?"

A. Adenocarcinoma (WRONG)
B. Carcinoid tumor
C. Hamartoma (CORRECT)
D. Small cell carcinoma
E. Squamos cell carcinoma

I know everyone is saying that "coin lesion" means its hamartoma but in first aid under lung cancer it says, "presentation cough, hemoptysis.... pneumonic "COIN" LESION on X-ray film or noncalcified nodule on CT". So why is adenocarcinoma right? Or why is hamartoma a better answer? Irregularly shaped glands seems like adenocarcinoma which has a glandular structure on histology?

"60 yo M 6 mo Hx of inability to sustain an erection, and decreased libido. For this period, he has also noticed he has had decreased energy, and a tendency to fall asleep by 7 pm. No Hx of depression, recent psychological stressors, or chronic pain. Significant physical changes since last visit 6 mo. ago: 2.54 cm (1 inch) loss of height & 7.62 (3 inches) increase in waist circumference> He also has gynecomastia & decreased muscle mass. Best drug to prescribe?
A. Epotein Alpha
B. Human Growth Hormone
C. Modanafil
D. Sildenafil (WRONG)
E. Testosterone (right?)

I'm just wondering what this guy has? Is it a pathological process or just a normal part of aging in men due to increased estrogen? I'm guessing the answer was testosterone due to his long list of hyperestrogenism symptoms. But given that he's >50 years old, I was thinking testosterone wouldn't be good for him due to higher incidence of BPH in his age group.


Thanks in Advance!

Members don't see this ad.
 

Attachments

  • NBME 11 question.PNG
    NBME 11 question.PNG
    43.4 KB · Views: 68
1. This question is terribly worded. The idea is that it is expressed more in males than females, and thus if the newborn were in fact female, then her brother would be at a GREATER risk of having pyloric stenosis. This is a stupid question though because the way it is worded makes it sound like the newborn has an older brother who will develop stenosis later in life, when I think it actually is referring to if the parents had another child who was a boy. Also, "fraternal twin" could be a sister -- fraternal twins can be different genders, which would mean a fraternal sister would have a LOWER risk.

2. You're overthinking this. What nerve controls pupillary constriction? CN3. If CN3 is broken on the left, then left can't constrict regardless of which eye you shine the light in. The edinger-westphal nucleus still works though CN3, as far as I remember. See this diagram: http://www.bioon.com/bioline/neurosci/course/cvis6.gif
Light travels in through the left eye (the bottom one in the diagram), travels back to the EW nucleus, which tells both eyes to constrict via CN 3. If CN3 is broken, then that eye cannot constrict.

3. It says "serum total cholesterol concentration," which implies a scaling value, not a binary measure. If the question were "Compared subjects who had a cholesterol over 200 to those below," then Chi-square would be right. As it is, though, you want to compare all along a spectrum, which means ANOVA is the correct choice.

4. She wants to burn 2 pounds in a week, at 3500 calories per pound = 7000 total calories needed to burn per week, or 1000 calories per day. She can burn this via decreased intake or increased exercise. She can either do 1000 calories worth of exercise per day, or eat 1000 calories less per day, or some combination of both. So your formula is (Time spent exercising)*500calories/hr+(Calorie deficit) = 1000. Not sure how to speed this process up, but once you know that each hour of exercise is worth 500 calories, and if you know you need to cut 1000 calories per day, you can go through the answers quickly and realize that none of them get close to the 1000calories/day except for the exercise 1 hour + cut by 500 option. The others are all short. This is a simple math problem, has almost nothing to do with physio except for knowing that weight loss can occur via decreased intake or via increased exercise.

5. Not sure on this one, I got it wrong. Sounded like an Adeno to me. Wiki says "Benign causes tend to have a well defined border, whereas lobulated lesions or those with an irregular margin extending into the neighbouring tissue tend to be malignant" -- I read invasion, I thought of cancer, and glands = Adeno.

6. He has testosterone deficiency. This is normal to some degree, but seems excessive in this patient. Sildenafil would only help him have+maintain an erection, it wouldn't help with any of the other symptoms. If the question were "An otherwise healthy male needs help maintaining an erection. Physical exam is unremarkable," then Sildenafil would be the answer. This guy, though, has many other things going on, all of which can be explained by his low testosterone. You are right about his risk of BPH (and more importantly, prostate cancer) being sensitive to testosterone supplementation, and would likely necessitate a workup before prescribing, but we're more worried about his current presentation than his future cancer risk.

Hope that was helpful.
 
Thank you for such a thorough reply. It was very helpful!

If anyone can explain the hamartoma question, then please help!

"54 y.o woman with 5 cm coin lesion in right upper lobe of lung. proliferation of irregularly shaped glands with hyperchromatic and pleomorphic nuclei invading the pulmonary parenchyma and lymphovascular spaces? Best described as?"

A. Adenocarcinoma (WRONG)
B. Carcinoid tumor
C. Hamartoma (CORRECT)
D. Small cell carcinoma
E. Squamos cell carcinoma
 
Members don't see this ad :)
Thank you for such a thorough reply. It was very helpful!

If anyone can explain the hamartoma question, then please help!

"54 y.o woman with 5 cm coin lesion in right upper lobe of lung. proliferation of irregularly shaped glands with hyperchromatic and pleomorphic nuclei invading the pulmonary parenchyma and lymphovascular spaces? Best described as?"

A. Adenocarcinoma (WRONG)
B. Carcinoid tumor
C. Hamartoma (CORRECT)
D. Small cell carcinoma
E. Squamos cell carcinoma

Adenocarcinoma is benign. The description describes a malignant lesion.
 
As far as the question that you have posted is concerned the answer is Adenocarcinoma.

Can you tell us the Block and question#??


Exam section 3: item 46

I just went back to check my answers and I saw that I accidentally selected "squamous cell carcinoma". That's probably why I had it wrong. Sorry for the confusion.

But even then, I've seen mixed posts on other threads between the answer being Adenocarcinoma vs Hamartoma? It would be great if you could confirm the answer.

Thanks!
 
Oh sorry I mistyped. Yeah that's interesting then but I could've sworn I got this right somehow and now I can't remember
 
Exam section 3: item 46

I just went back to check my answers and I saw that I accidentally selected "squamous cell carcinoma". That's probably why I had it wrong. Sorry for the confusion.

But even then, I've seen mixed posts on other threads between the answer being Adenocarcinoma vs Hamartoma? It would be great if you could confirm the answer.

Thanks!

Just checked Adenocarcinoma is right. I almost got tricked in to carcinoid but I figured there had to be more history. If you look up NBME 11 Block 2 Discussion in google, you'll find the explanation I believe its question #42 in the forum :)
 
Just checked Adenocarcinoma is right. I almost got tricked in to carcinoid but I figured there had to be more history. If you look up NBME 11 Block 2 Discussion in google, you'll find the explanation I believe its question #42 in the forum :)

Thank you so much! I can't find the page that you referenced. Do you have the link? Can you post it?
 
Top